2014 dxdy logo

Научный форум dxdy

Математика, Физика, Computer Science, Machine Learning, LaTeX, Механика и Техника, Химия,
Биология и Медицина, Экономика и Финансовая Математика, Гуманитарные науки




Начать новую тему Ответить на тему На страницу 1, 2, 3, 4  След.
 
 Две задачи про сопротивления
Сообщение12.12.2019, 19:42 
Заслуженный участник


10/01/16
2318
1. Имеется электрическая цепь (т.е., связный граф, ребра которого суть резисторы, с разным, вообще говоря, сопротивлением). Измеряем сопротивление между вершинами $A$ и $B$ цепи. Показать, что при удалении одного из ребер, сопротивление не уменьшится.

(Оффтоп)

Задача известная, а вспомнилось в связи с задачей 6 с Олимпиады Лобачевского-2019, когда здравый смысл говорит "ну конечно", а с точным решением - проблемы...

2. Найти сопротивление между соседними узлами бесконечной цепи вида "правильная квадратная решетка", если сопротивление каждого "ребрышка" цепи равно 1 Оммм...
Варианты: для других правильных решеток (треугольной и шестиугольной).

(Оффтоп)

Задачи 1, 2 мне известны со стародавних времен, из устных преданий. Однако, честно признаюсь: решать их я не умею. Так что не уверен в правильности выбора раздела для этой темы.

 Профиль  
                  
 
 Re: Две задачи про сопротивления
Сообщение12.12.2019, 19:48 
Заслуженный участник
Аватара пользователя


01/09/13
4676
В первой задаче, видимо, так же имеется в виду бесконечный граф?

 Профиль  
                  
 
 Re: Две задачи про сопротивления
Сообщение12.12.2019, 20:03 
Заслуженный участник
Аватара пользователя


30/01/06
72407
DeBill в сообщении #1429905 писал(а):
2. Найти сопротивление между соседними узлами бесконечной цепи вида "правильная квадратная решетка", если сопротивление каждого "ребрышка" цепи равно 1 Оммм...
Варианты: для других правильных решеток (треугольной и шестиугольной).

Буквально несколько дней назад меня заинтересовал этот вопрос для других пар узлов.

Для соседних узлов задача решается элементарно и по-школьному. А уже даже для диагональных (ответ $\tfrac{2}{\pi}$ - при чём тут пи???) - только с помощью суровой высшей математики. И вообще мало где про это, оказывается, написано.

С Physics.SE https://physics.stackexchange.com/questions/2072/on-this-infinite-grid-of-resistors-whats-the-equivalent-resistance
меня отослали на вот этот сайт, на котором несколько "заходов" на эту задачу:

Я одолел только первую часть, с конечно-разностными уравнениями, и тоже не во всём разобрался. Но задача явно непростая и интересная.

-- 12.12.2019 20:04:16 --

Geen в сообщении #1429907 писал(а):
В первой задаче, видимо, так же имеется в виду бесконечный граф?

Нет, любой. Это тоже интересная теорема.

 Профиль  
                  
 
 Re: Две задачи про сопротивления
Сообщение12.12.2019, 21:19 
Заслуженный участник
Аватара пользователя


04/09/14
5288
ФТИ им. Иоффе СПб
DeBill в сообщении #1429905 писал(а):
Измеряем сопротивление между вершинами $A$ и $B$ цепи. Показать, что при удалении одного из ребер, сопротивление не уменьшится.
Ответ: "Ток течет так, что бы минимизировать джоулево тепло" не катит?

 Профиль  
                  
 
 Re: Две задачи про сопротивления
Сообщение12.12.2019, 21:50 
Заслуженный участник


10/01/16
2318
amon в сообщении #1429915 писал(а):
Ответ: "Ток течет так, что бы минимизировать джоулево тепло" не катит?

Ну, может, и катит. Но как это формализовать?
Т.е., хотелось бы иметь решение типа: вот у нас есть система уравнений, составленная по правилам Киркгофа.
Вот - другая система, полученная из первой заменой одного из коэффиентов (сопротивлений) на больший. Тогда суммарный ток из $A$ для второй системы не больше чем для первой....
А в идеале - решение, которое можно впарить школьнику.

 Профиль  
                  
 
 Re: Две задачи про сопротивления
Сообщение12.12.2019, 22:08 
Заслуженный участник
Аватара пользователя


04/09/14
5288
ФТИ им. Иоффе СПб
DeBill в сообщении #1429920 писал(а):
Но как это формализовать?
На физическом уровне строгости примерно так. Подключим к цепи генератор тока. Ток распределиться так, что бы выделяемое тепло было минимальным (как это доказывать школьнику - я пас). Это тепло равно $I^2R.$ Теперь перережем одну проволочку. Число способов току перераспределиться точно не увеличилось, значит сопротивление не уменьшилось - иначе уменьшится выделяемое тепло, а оно и так было минимально.

 Профиль  
                  
 
 Re: Две задачи про сопротивления
Сообщение12.12.2019, 22:14 
Заслуженный участник
Аватара пользователя


23/07/08
10910
Crna Gora
amon в сообщении #1429922 писал(а):
Ток распределиться так, что бы выделяемое тепло было минимальным
Некогда активный участник obar предлагал эту задачу в Олимпиадных задачах (Ф):
obar в сообщении #524503 писал(а):
1. Показать, что в произвольной электрической цепи, состоящей лишь из резисторов, распределение токов такое, что рассеиваемая суммарная тепловая мощность минимальна (при фиксированном полном токе).
Для удобства повторю своё решение здесь:

Выберем для каждого резистора положительное направление протекания тока. Пусть через резистор $R_k$ течет ток $I_k$.
Теперь добавим дополнительные контурные токи $J_m$, где $m$ -- номер контура. Тем самым закон Кирхгофа для токов продолжает выполняться (и сохраняется полный ток), а для напряжений -- нарушается. Пусть в результате через резистор $R_k$ течет ток $I_k+\Delta I_k$. Тогда суммарная мощность изменится на величину$$2\sum\limits_k R_k I_k \Delta I_k + \sum\limits_k R_k (\Delta I_k)^2$$Вторая сумма положительна (коль хоть на одном резисторе ток изменился). Докажем, что первая сумма равна нулю -- что и даст решение задачи.

Заметим, что $\Delta I_k=\sum\limits_m\varepsilon_{km} J_m$, где $\varepsilon_{km}$ равно
$0$, если $m$-й контур не проходит через $k$-й резистор,
$+1, -1$, если он-таки проходит, и выбранное положительное направление тока $m$-го контура совпадает/противоположно положительному направлению $k$-го резистора.

Подставим $\Delta I_k$ в первую сумму:
$$\sum\limits_k R_k I_k \Delta I_k = \sum\limits_m \left(J_m \sum\limits_k R_k I_k \varepsilon_{km}\right)$$Внутренняя сумма -- это сумма падений напряжений по $m$-му контуру, а так как для исходных токов $I_k$ закон напряжений Кирхгофа выполнялся, она равна нулю.

 Профиль  
                  
 
 Re: Две задачи про сопротивления
Сообщение12.12.2019, 22:25 
Заслуженный участник
Аватара пользователя


30/01/06
72407
amon в сообщении #1429922 писал(а):
Ток распределится так, чтобы выделяемое тепло было минимальным (как это доказывать школьнику - я пас).

Я и это правило искал недавно... и нашёл, на свою голову, аж в двух версиях. В одной к цепи подключён источник напряжения, в другой - источник тока.
Цедулька - вот она, кстати, свеженькая, этогодняшняя, индус какой-то писал, хотя и из Пердью:

 Профиль  
                  
 
 Re: Две задачи про сопротивления
Сообщение12.12.2019, 22:42 
Заслуженный участник


10/01/16
2318
svv
Круто!
Вот пытаюсь еще осознать это чисто математически...

 Профиль  
                  
 
 Re: Две задачи про сопротивления
Сообщение12.12.2019, 23:10 
Заслуженный участник
Аватара пользователя


23/07/08
10910
Crna Gora
К сожалению, это не решение собственно Вашей задачи 1, это только обоснование утверждения amon.

del. Отказываюсь от идеи доказать на основе теоремы взаимности.

 Профиль  
                  
 
 Re: Две задачи про сопротивления
Сообщение13.12.2019, 00:11 
Аватара пользователя


11/12/16
14036
уездный город Н
Пассивный четырехполюсник может быть заменен эквивалентной Т- или П-схемой замещения.

 Профиль  
                  
 
 Re: Две задачи про сопротивления
Сообщение13.12.2019, 17:17 


27/08/16
10452
Munin в сообщении #1429928 писал(а):
Я и это правило искал недавно...
Насколько я понимаю, это совершенно тривиальное следствие совершенно нетривиального утверждения из необратимой термодинамики, что скорость производства энтропии в термодинамической системе с линейными феноменологическими соотношениями, обратимой на микроуровне, в стационарном состоянии минимальна. Т. е при наличии магнитного поля в сплошной среде это уже бывает не так: дополнительная прорезь может уменьшить сопротивление образца.

 Профиль  
                  
 
 Re: Две задачи про сопротивления
Сообщение13.12.2019, 20:53 
Заслуженный участник
Аватара пользователя


04/09/14
5288
ФТИ им. Иоффе СПб
realeugene в сообщении #1429995 писал(а):
это совершенно тривиальное следствие совершенно нетривиального утверждения из необратимой термодинамики
Ну, термодинамика, по большому счету, тут и не нужна. Достаточно трех формул
\begin{align}
\mathbf{j}&=\sigma \mathbf{E}\\
\operatorname{div}\mathbf{j}&=0\\
\operatorname{rot}\mathbf{E}&=0
\end{align}

 Профиль  
                  
 
 Re: Две задачи про сопротивления
Сообщение13.12.2019, 23:59 


27/08/16
10452
amon,
2 и 3 - это уравнения Максвелла и обратимы. А вот 1 генерирует энтропию и, поэтому, термодинамика с применимостью теоремы Онзагера. В магнитном поле закон Ома нарушается эффектом Холла.

Среда может быть неоднородной и неизотропной, но пока на микроуровне она обратима, утверждение про минимум генерации энтропии в стационарном процессе сохраняется. Т. е. резисторы в цепи с сосредоточенными параметрами могут быть разных типов и номиналов.

 Профиль  
                  
 
 Re: Две задачи про сопротивления
Сообщение19.12.2019, 13:20 
Модератор
Аватара пользователя


11/01/06
5710
Ещё задачка по теме.

Докажите, что для любых положительных чисел $a,b,c,d$ выполнено неравенство:
$$\frac{(a+c)(b+d)}{a+b+c+d} \geq \frac{ab}{a+b} + \frac{cd}{c+d}.$$

 Профиль  
                  
Показать сообщения за:  Поле сортировки  
Начать новую тему Ответить на тему  [ Сообщений: 50 ]  На страницу 1, 2, 3, 4  След.

Модераторы: Модераторы Математики, Супермодераторы



Кто сейчас на конференции

Сейчас этот форум просматривают: YandexBot [bot]


Вы не можете начинать темы
Вы не можете отвечать на сообщения
Вы не можете редактировать свои сообщения
Вы не можете удалять свои сообщения
Вы не можете добавлять вложения

Найти:
Powered by phpBB © 2000, 2002, 2005, 2007 phpBB Group